Floor equation

Sdílet
Vložit
  • čas přidán 15. 02. 2024
  • In this video , I showed how solve a simple floor equation

Komentáře • 229

  • @JSSTyger
    @JSSTyger Před 4 měsíci +305

    I took 10 university math courses but never learned floor/ceiling functions. Thanks for enlightening me.

    • @mohammedel-gamal3455
      @mohammedel-gamal3455 Před 4 měsíci +44

      same here,
      never stop learning. those who stop learning , stop living

    • @Mustapha.Math_at_KUSTWUDIL
      @Mustapha.Math_at_KUSTWUDIL Před 4 měsíci +11

      Another name of this function is greatest integer function.

    • @thelightningwave
      @thelightningwave Před 4 měsíci +26

      This my surprise you, but you were already were using it, when ever people talk about their age they always give an integer, and that integer is the floor function of their actual age.

    • @m.h.6470
      @m.h.6470 Před 4 měsíci +6

      @@thelightningwave not really, it is more like the rounding function. If you are less than a month away from your birthday, you typically don't say your "old" age, you say "I'm going to be X next month".

    • @NekoChan_TV
      @NekoChan_TV Před 4 měsíci

      That’s the triad from R to Z : floor, cuisine and round :)

  • @rinaldogarcia7759
    @rinaldogarcia7759 Před 3 měsíci +91

    this is so calming and soothing in some way... the clacking of the chalk on the blackboard, him speaking so quietly and calmly... I love these videos, great work!

    • @QFredfons
      @QFredfons Před 3 měsíci

      The quality of his voice also helps. It's soothing but motivational!

  • @boristhedestroyerofancient508
    @boristhedestroyerofancient508 Před 4 měsíci +68

    Man, you really have the most relaxing style of teaching maths ever 😅. I like the hyper-focused youtubers as well but your videos make me not want to click off with how peaceful they are.

  • @corneliusagu2903
    @corneliusagu2903 Před 4 měsíci +48

    I have got to the peak of my studies having has a PhD. in Process and energy engineering. On my love for mathematics, which controls my daily activities at work, I often spend time looking at tutors online for basic mathematics. I have been following your videos, and at every point in time and I have developed different interests in your teaching approach beside that I learn new things like this today which I have not come across. Well done, brother, and I urge you to continue at this pace.

    • @valiant8987
      @valiant8987 Před 4 měsíci +2

      He is far beyond a basic math tutor

  • @RoyBoy_i3c
    @RoyBoy_i3c Před 4 měsíci +10

    He helps me not only learn of new functions, but also helps me fall asleep at night. Keep it up!

  • @MichaelAdjei-up2ce
    @MichaelAdjei-up2ce Před 4 měsíci +15

    What an amazing teacher.
    This channel has become my favorite now ❤🎉

  • @ounaogot
    @ounaogot Před 4 měsíci +10

    I like your teaching style, am slowly incorporating it into my online classes, so well explained always, NEVER STOP LEARNING....

  • @magefreak9356
    @magefreak9356 Před 4 měsíci +15

    Great video as always!
    It would have been cool to have seen graphs of both the floor(3x) and floor(x) and see where the sum of these adds up to 6.
    Never stop learning

    • @fahrenheit2101
      @fahrenheit2101 Před 3 měsíci +4

      Think about it:
      floor(x) is integer valued, and steps up by 1 at each next integer, starting with floor(0) = 0
      floor(3x) is similar, but steps up by 1 at each 1/3, starting again at floor(3*0) = 0
      So the function on the LHS starts at f(0) = 0, but steps up by 1 at every integer, and also by 1 at every 1/3 : the combined effect is a step of 1 at ever 1/3, except every 3/3 = 1, i.e. every integer step, for which it jumps up by two.
      Long story short, it starts at 0, and takes jumps of size 1,1,2,1,1,2,1,1,2... etc., at every 1/3.
      That tells you what it looks like and if you cut off the steps after 1+1+2+1+1 = 6, then you've taken 5 steps of size 1/3 so you're at 5/3.
      So we need the bit of the function starting at 5/3 and going right up until 6/3 = 2, but not including that endpoint.
      Hopefully that made some sense.

  • @danobro
    @danobro Před 4 měsíci +15

    Wow. six views and six likes. Everyone liked! (not surprised, love your vids)

  • @TheRealZeaga
    @TheRealZeaga Před 3 měsíci +3

    You're my new favorite math channel. Your voice is so calming! Thank you for the video :)

  • @IITianNikhil
    @IITianNikhil Před 4 měsíci +33

    The basics that you teach in your videos are the pillars of India's most difficult exam like JEE Advance.

    • @SakshamMahajanB
      @SakshamMahajanB Před 4 měsíci

      Are you a IITIAN?? Or a aspirant?

    • @ts9dream
      @ts9dream Před 4 měsíci

      the question is does it really matter? he never said this video is intended for a jee aspirant

    • @SakshamMahajanB
      @SakshamMahajanB Před 4 měsíci +6

      @@ts9dream why are you concerned for that I just want to ask him whether he is a IITIAN/nitian or aspirant if he were a IITIAN I would ask my question . Who the hell are you concerned with that

  • @harshit7510
    @harshit7510 Před 3 měsíci +1

    There's also a very good intuitive way of solving it
    By hit and trial
    I put x=1
    3+1 = 4 6
    So by analysing this
    1

  • @ReallyAmateurPianist
    @ReallyAmateurPianist Před 3 měsíci +3

    Nice video!
    My solution for this was:
    Left hand side is 4 with x=1, and 8 with x=2. Therefore we know that 1

    • @fahrenheit2101
      @fahrenheit2101 Před 3 měsíci

      Yep, what I did too. Another idea I had which is a little less formal is to just literally picture the function. The interesting stuff happens every 1/3, and it's just an increment by 1 each time, except at the integer values of x where you get an increment of 2 (I'm sure you can see why). Tracking that, you see that you get 6 after 5 steps, hence 5/3 up to but not including 6/3 = 2.
      This also has the benefit of telling you that certain values are unattainable by this function (specifically anything congruent to 3 mod 4)

  • @edvardm4348
    @edvardm4348 Před 3 měsíci +6

    Subscribed. Who could not appreciate the style of teaching here? I'm huge fan of more "traditional" math videos like Mathologer and 3b1b, but following this guy is like being taken to an adventure, or detective story. Just love this

  • @lesheq85
    @lesheq85 Před 3 měsíci +2

    i love how happy you are explaining this. even tho i was familiar with floor and ceiling from programming i have never solved equality containing them. thanks for that!

  • @akultechz2342
    @akultechz2342 Před 4 měsíci +4

    Being a jee advanced aspirant this looked easy tho.
    Taking x = I + F
    3I + [3F] + I = 6
    Now 4I + [3F] = 6
    Here, I must be >0
    So I = 1 satisfies only.
    Now we have [3F] = 2
    Let 3F = t
    Then t belongs to [2,3)
    So F belongs to [⅔,1)
    Hence x belongs to [5/3,2)

    • @Mothuzad
      @Mothuzad Před 4 měsíci

      This was my approach as well. I named my variables n and u, but that hardly matters.

    • @melikmourali2072
      @melikmourali2072 Před 3 měsíci

      The thing missing in your proof is that you have to prove that x is in [1,2).
      How I did it is that I plugged x=1 and x=2 and used the fact that the floor value function is increasing and that is easy to prove as well, if you have to. (It was implied in your reasoning but you have to show it.) Prime Newtons' proof is cleaner though I must admit.

  • @vitotozzi1972
    @vitotozzi1972 Před 4 měsíci +4

    Your explain is perfect!!! Congratulation!

  • @JourneyThroughMath
    @JourneyThroughMath Před 4 měsíci +7

    Dang, I made some faulty assumptions when I attempted this the first time. Great video as always!

    • @jumpman8282
      @jumpman8282 Před 4 měsíci

      Let me guess, floor(3x) + floor(x) = floor(4x) ?

    • @JourneyThroughMath
      @JourneyThroughMath Před 4 měsíci +2

      @@jumpman8282 nope, i wanted to bring the 3 to the outsideof the floor, like one would with |3x|

    • @DatBoi_TheGudBIAS
      @DatBoi_TheGudBIAS Před 4 měsíci +2

      ​@@JourneyThroughMathye, I had the same idea, but then I tried it with some values, and saw it wasn't true. Specifically 0.4
      floor(3*0.4)=floor(1.2)=1
      3floor(0.4)=0

    • @JourneyThroughMath
      @JourneyThroughMath Před 4 měsíci +1

      Yeah, I havea bad habit of not doing that. It occured to me after I should, but Ill probably do it again one day

  • @seedmole
    @seedmole Před 3 měsíci

    Your explanation in the introduction was good enough for me to quickly solve it in the Pure Data graphing/programming environment. Great video!

  • @east9876
    @east9876 Před 3 měsíci

    As a programmer I've grown quite familiar with using floor and ceiling functions. But despite years of math and computer science classes I've never come across such a simple yet in depth explanation of the mathametical properties behind them. Excellent video! Looking forward to checking out what else your channel has to offer

  • @user-kj7hr3qw9w
    @user-kj7hr3qw9w Před 3 měsíci

    Your style is very relaxed. I didn't think mathematics could be taught this way
    THNX❤

  • @NekoChan_TV
    @NekoChan_TV Před 4 měsíci +5

    12:25 in France we use different notations for that, it's great to know other notations !
    We don't write [5/3, 2) in France, but [5/3; 2[ with bracket facing outwards for excluding value :)
    btw I dropped a subscribe here !

    • @PrimeNewtons
      @PrimeNewtons  Před 4 měsíci +5

      That's new to me. Interesting to discover 🤔

    • @m.h.6470
      @m.h.6470 Před 4 měsíci +4

      In Germany we learn both ways. It is up to the teacher, which one you are supposed to use...

    • @NekoChan_TV
      @NekoChan_TV Před 4 měsíci +1

      we also have this notation [[3; 7]] for integer interval :)

    • @user-sl7ie9te5r
      @user-sl7ie9te5r Před 4 měsíci +1

      I've seen the Portuguese use outward-facing parentheses
      Poles use something like >

    • @Pandorarl
      @Pandorarl Před 3 měsíci

      in norway we would write [5/3,2> big > though

  • @ReyazulislamReayal
    @ReyazulislamReayal Před 4 měsíci +17

    You are a grate teacher❤❤❤

  • @aadityavikram5030
    @aadityavikram5030 Před 4 měsíci +3

    Thanks for considering my request and teaching another question of this type....

  • @aryahan4976
    @aryahan4976 Před 4 měsíci +1

    nice, the problem solved beautifully and as well as u

  • @Niki1A_
    @Niki1A_ Před 3 měsíci +1

    I got to the solution much quicker looking at the thumbnail by observing that it is a (non-strictly) monotonely rising function, so you just have to find the minimum and supremum of it working (minimum and supremum because the floor function includes the lower bound, but excludes the upper bound). There can't be any negatives involved because both terms would be negative then. Also 2 is clearly at least an upper bound. Also both floor terms must be integers, due to the floor function. Therefore, you only have to consider how to split up the 6 into two non negative integers:
    6+0 doesn't work because for floor(3x) = 6, you would need x >= 2, but then floor(x)>=2>0.
    5+1 does work with floor(3x)=5 giving us x>=5/3 as a lower bound and floor(x)=1 giving us x

  • @AmeNoTenshi
    @AmeNoTenshi Před 2 dny +1

    that smile always gets me xDD
    thank you so much for the simplification
    great work

  • @hvok99
    @hvok99 Před 4 měsíci

    Wow, very nice proof method. Loved the s
    Work with systems of compound inequalities. Would love to see more problem solving with compound inequalities!! Very cool.

  • @crazypantaloons
    @crazypantaloons Před 3 měsíci

    This problem has a trivial solution. I think the video solution glosses over the best property of floor math: the floor of an integer is itself. Find the integers, and remove them.
    Step 1: on quick inspection, x must be between 1 and 2, so let x = 1 + y (where 0 < y < 1), then floor (3x) + floor (x) = 6 reduces to 3 + floor (3y) + 1 + floor(y) = 6, or floor (3y) + floor (y) = 2.
    Step 2: we purposely picked y such that floor (y) is 0, so floor (3y) = 2, and 2/3

  • @duckyoutube6318
    @duckyoutube6318 Před 4 měsíci +2

    The meaning of floor is less than or equal to.
    Thank you Mr.Prime. Im not in college but when i get there.... ima pass calc 3 on day 1.

  • @Mr.Rinfinite
    @Mr.Rinfinite Před 3 měsíci +1

    An interesting thing I thought of, this means the equation:
    floor(3x) + floor(x) = 7
    Actually has no solutions!
    Because if x=2, our RHS turns to 8, but anything up to 2 (1.99999 for example) would still have our RHS being 6.
    You can get the equation's RHS to equal 4 (x=1) and also 5 in a range of [4/3 , 5/3), as well as 6 and 8, as you showed, but never 7. Kind of interesting!

  • @duckyoutube6318
    @duckyoutube6318 Před 4 měsíci +4

    The floor of x?!
    New math 😮 how very exciting.

  • @editvega803
    @editvega803 Před 3 měsíci +1

    You are a great teacher! 👏

  • @luiscrispinvargas3061

    Muchas gracias por el vídeo, acabo de aprender una nueva forma de ver el ejercicio, saludos desde Perú.

  • @aquss33
    @aquss33 Před 4 měsíci +1

    I ain't never heard of these, you really make some interesting videos.

  • @rutgerdeboom7424
    @rutgerdeboom7424 Před měsícem

    You’re my favorite math youtuber :).

  • @antonionavarro1000
    @antonionavarro1000 Před 4 měsíci

    Magnífico tutorial. Vídeos muy didácticos, claros, sencillos y que van al núcleo del problema. Buen trabajo con el uso de la función 'floor'. La función 'ceiling' debe funcionar de manera similar. ¿Algún vídeo previsto sobre ella?

  • @TheFinagle
    @TheFinagle Před 3 měsíci

    And this video reminded me why I didn't like grade school math, even though I do actually love math. I figured this one out in my head in a few seconds (kinda using a limit like approach) BUT in grade school teachers expect to show ALL this work to get full marks even if you didn't need it.

  • @Nyambenyambe-xc9ib
    @Nyambenyambe-xc9ib Před 4 měsíci

    i love you man, thank you so much for what you are doing you really helping.

  • @kragiharp
    @kragiharp Před 4 měsíci +1

    Thanks a lot for this video and all the others!
    If you go on with them then soon we will all be math professors. 😊
    ❤️🙏

  • @jennymarx9228
    @jennymarx9228 Před 4 měsíci +2

    Your videos are really useful for me hats off brother 😌✨ (i'm in my 12th grade gonna face my exams soon🎉)

  • @lamug
    @lamug Před 4 měsíci

    Great video! Very well explained

  • @futellab
    @futellab Před 4 měsíci

    Oh my goodness precious ❤ he's so calm and he has mathematics. Dude I'm so much obsessed with his teaching style.... I hope if he will professor of my dream IIT or mit ❤

  • @MaxPicAxe
    @MaxPicAxe Před 3 měsíci +1

    You're my new favourite math CZcamsr

  • @jacobgoldman5780
    @jacobgoldman5780 Před 4 měsíci +1

    Before watching the video, x clearly cannot be an integer as that would mean 4x=6 or x=1.5 which is not an integer therefore a contradiction. Since the coefficient of one of the terms is 3 we should set x=n+(r/3) with 0

  • @andreaparma7201
    @andreaparma7201 Před 18 dny

    Let floor(x)=k, so that k

  • @cparks1000000
    @cparks1000000 Před 3 měsíci

    I'd just break the problem into cases. Every real number x can be written in the form x=k + r where r is between 0 and 1 (possibly equal to zero). We then break into 3 cases: r= 2/3, or neither. Case 1 is immediately ruled out because this implies that 4k=6. Similarly, case 3 implies 4k=5. This leaves case 2 as being the only possible. In case 2, we have k=1, so x is in [5/3, 2).
    Alternatively, you can use the fact that floor(x+k) = floor(x) + k for any integer. Thus
    floor(3x) + floor(x) =
    floor(3k + 3r) + k =
    3k + floor(3r) + k =
    4k + floor(3r). Using the fact that r is in [0,1), we know that floor(3r) can only be 0, 1, or 2. One then concludes that floor(3r) must be 2 and proceeded to calculate x.

  • @m-yday
    @m-yday Před 3 měsíci

    I really like how you teach!!

  • @HenrikMyrhaug
    @HenrikMyrhaug Před 3 měsíci

    Very interesting problem. I have never had to work with math on rounding functions like floor or ceiling, but this is a good intro.

  • @a.lollipop
    @a.lollipop Před 3 měsíci +1

    Never dealt with an equation including a floor yet, so I tried solving myself before watching. I did the following:
    first i noticed there will definetely be a range of values, so I should try to find a maximum and a minimum value that satisfies the equation so i know the range.
    i started by assuming x is an integer (i tried writing that the fancy way but there is no symbol for the set of integer numbers or for 'exists' on mobile :c), and realized 1 is too small and 2 is too big, therefore 1

    • @a.lollipop
      @a.lollipop Před 3 měsíci

      TIL editing a hearted comment removes the heart. Whoops.

    • @PrimeNewtons
      @PrimeNewtons  Před 3 měsíci +1

      That was good reasoning.

  • @whebon7266
    @whebon7266 Před 3 měsíci

    Beautiful handwriting!

  • @florianbasier
    @florianbasier Před 2 měsíci +1

    you took the long scenic road my friend. We know that no matter what x is, there is a unique (n, k, y) triplet from Nx{0,1,2}x[0,1/3[ so that x=n+k/3+y. Floor(3x)=Floor(3n+k+3y). 3n+k is a natural number and 0

    • @PrimeNewtons
      @PrimeNewtons  Před 2 měsíci

      I've seen comments saying something similar. I have to digest it and see how I can tell viewers to consistently look for such pattern in all similar problems. Thanks 😊

  • @sfinford
    @sfinford Před 4 měsíci +6

    as someone who paused and managed to solve it looking at it for less than 15 seconds, i feel very accomplished

  • @sudonim116
    @sudonim116 Před 4 měsíci

    I love your channel, makes me bored in my normal maths lessons though haha

  • @orang1921
    @orang1921 Před 4 měsíci

    fantastic video

  • @kacperkrul
    @kacperkrul Před 3 měsíci

    This is pretty cool, never seen a floor equation before

  • @BorisNVM
    @BorisNVM Před 3 měsíci

    For these equations I always write
    x = n + a
    where
    n integer
    0

  • @Dominus_Potatus
    @Dominus_Potatus Před 4 měsíci +2

    I love this one since it is not even taught when I was in school

  • @estebansalgado4708
    @estebansalgado4708 Před 4 měsíci +1

    I took a slightly different approach. I wrote x as the sum of its integer part k and its fractional part f. Then the equation becomes
    4k + floor(3f) = 6
    Given that 0

    • @fahrenheit2101
      @fahrenheit2101 Před 3 měsíci

      Nice systematic approach. Not at all what I went for, but definitely one I'll bear in mind.

  • @shadrackshija2848
    @shadrackshija2848 Před 3 měsíci

    Thank you for showing the way on how to go about

  • @rssl5500
    @rssl5500 Před 4 měsíci

    I’m in 11th grade and we have these problems very frequently especially in competition math problems but still we solve this differently
    Notice :
    [kx]=[x]+[x+1/k]+…+[x+k-1/k]
    where k is an integer
    Hence :
    [x]+[x]+[x+1/3]+[x+2/3]=6
    notice if x = [x]+ p then we can partition this into the following cases :
    1: 0

  • @axh759
    @axh759 Před 2 měsíci

    Keep going

  • @xadxtya
    @xadxtya Před 4 měsíci

    You're great!

  • @bogusawsroda3747
    @bogusawsroda3747 Před 4 měsíci +2

    I like your personality

  • @pizza8725
    @pizza8725 Před 4 měsíci +1

    I calculated in my mind in like 10 seconds that x would be at least 5÷3 and smaller than 6

  • @ibrahem_x564
    @ibrahem_x564 Před 4 měsíci +1

    Thanks so much . ❤❤

  • @hydropage2855
    @hydropage2855 Před 3 měsíci +2

    Bob ross of math

  • @tjipondauheua453
    @tjipondauheua453 Před měsícem +1

    when it comes to the equation formation is that supposed to be 3t instead of letting it be t since tis representing 3x

  • @pvanukoff
    @pvanukoff Před 3 měsíci

    The Bob Ross of math right here.

  • @Icephoenix0344
    @Icephoenix0344 Před 2 měsíci +1

    Hi, I saw a nice equation from the national math olympics from germany: What x satisfys floor(x/2) * floor(x/3) * floor(x/4) = x^2? I didnt ever see a floor equation where you have a division in the floor operater, so I would really enjoy a video about it.

    • @PrimeNewtons
      @PrimeNewtons  Před 2 měsíci

      Interesting! I already have 3 solutions, but I need to see the original problem to know what type and how many solutions I need. Please email to me. Primenewtons@gmail.com. Thank you.

    • @Icephoenix0344
      @Icephoenix0344 Před 2 měsíci

      @@PrimeNewtons Thank you for your reply, I sent you an Email under the caption Math Problem.

    • @PrimeNewtons
      @PrimeNewtons  Před 2 měsíci

      Please check again. I didn't get your email.

    • @Icephoenix0344
      @Icephoenix0344 Před 2 měsíci

      @@PrimeNewtons I sent it again

  • @janmesh2332
    @janmesh2332 Před 3 měsíci +1

    This is so cool

  • @fahrenheit2101
    @fahrenheit2101 Před 3 měsíci

    Haven't watched it, but here's my quick answer:
    We can exclude solutions greater or equal to 2 for obvious reasons: floor is an increasing function, and floor(3*2) + floor(2) = 8 so for x = 2 and beyond, the LHS is at least 8
    Similar reasoning excludes solutions less than or equal to x = 1.
    This guarantees that floor(x) = 1, and we only need floor(3x) = 5, which gives the inequality 5

  • @spikedskull137
    @spikedskull137 Před 3 měsíci

    I would never think that you can solve these kinds of problems

  • @jamesbakis6330
    @jamesbakis6330 Před 4 měsíci +1

    i love this guy

  • @varno
    @varno Před 3 měsíci

    There is a simpler way to see the first part here.
    The function is monotonic increasing. Further, the function is based on the floor value, so it only changes when either 3x is an integer or x is an integer, which coincide on the fractions k/3, being the same on half-open intervals. Further, on these critical values this function is no greater than the equality of the linearised version of the function I.e. 4x=6, which can be solved as x=1.5/2 thus we only have to test the critical points and above 1.5, these are, 5/3 and 6/3, as the functions are equal on the integers.
    As the 5/3 value equals 6 and the 6/3 value is greater then this function must be equal to 6 over the half open interval [5/3,6/3=2)

    • @fahrenheit2101
      @fahrenheit2101 Před 3 měsíci

      You lost me when you said "linear function 4x = 6" which isn't true. That's an equation, only the LHS is a function. You might say "you know what I mean" but... I actually don't. You're probably right in what you're saying, I'd just appreciate some precision. The function in the question is no greater than the function 4x. It doesn't make sense to be no greater than 4x = 6...
      But you can conclude that since the function is at most 4x, and 4x = 6 at x = 1.5, we can rule out x < 1.5 on those grounds. Which is probably what you meant, but not even close to what you said.

  • @omendrakumar3416
    @omendrakumar3416 Před 3 měsíci +1

    It is a part of Greatest integer function

  • @OLEGEK23
    @OLEGEK23 Před 4 měsíci +2

    Отличный пример!

  • @morsilimohamed9354
    @morsilimohamed9354 Před 3 měsíci

    Good job

  • @Dalroc
    @Dalroc Před 3 měsíci

    3x+x>6 if x>=2
    3x+x

  • @freya28733
    @freya28733 Před 4 měsíci +2

    it's a very rigorous proof but imo way too overkill, is this done on purpose to teach how to work with more complicated floor functions or is this how you would solve it if you just wanted an awnser? no hate btw just curious

  • @user-tw5zf7gs2r
    @user-tw5zf7gs2r Před 3 měsíci

    Great teaching!
    but i noticed that in 4:58 you could add the sections in both equations to get rid of k instently, and you are only left with x

    • @PrimeNewtons
      @PrimeNewtons  Před 3 měsíci

      You'd have to add k to 3x too

    • @user-tw5zf7gs2r
      @user-tw5zf7gs2r Před 3 měsíci

      @@PrimeNewtons no i ment adding the equations together
      Adding the x on the top and 3x on the bottom for example, and on the right and left you have +k on the top and -k on the bottom that will cancel out

    • @fahrenheit2101
      @fahrenheit2101 Před 3 měsíci +1

      Yes, but this doesn't fully solve the equation, it gives an interval that's too large.
      Of course it's still 100% true and valid to say x lies in that interval, it's just not the solution set.

  • @redmask6085
    @redmask6085 Před 4 měsíci

    So before watching the video I tried solving it myself and got the same solution, but I used a diffrent method (also I'm going to use E as 'is contained in'):
    first I declered x=k+d, k E Z, d E [0,1) => floor(x)=k
    and after substitiusion I got:
    floor(3(k+d))+k=6
    floor(3k+3d)+k=6 now k E Z => 3k E Z => floor(3k+a)=3k+floor(a)
    as the only thing that the floor changes is the >=0,

  • @B_u_L_i
    @B_u_L_i Před 4 měsíci

    Came for the floor equation, stayed for the asmr chalk on board sounds

  • @1ne2woPandemonium
    @1ne2woPandemonium Před 4 měsíci

    Hush! A lil bit of maths still makes me happy😄

  • @paulor.r.correia1789
    @paulor.r.correia1789 Před měsícem

    Excelente. Excelent. 🇧🇷🇧🇷🇧🇷🇧🇷

  • @user-pv5hd1vu1t
    @user-pv5hd1vu1t Před 3 měsíci

    The way I did it.
    If x was an integer
    3x + x = 6
    => 4x = 6
    => x = 6/4
    => x = 3/2
    but 3/2 is not an integer
    so x can't be an integer.
    3/2 = 1.5
    So as a guess, check x \in [1,2) since we are getting x = 1.5 from 'integer contradiction'
    Let x = 1 + epsilon where epsilon \in (0,1)
    This means
    3x = 3 + 3epsilon
    floor(3x) = floor(3 + 3epsilon)
    Notice 3epsilon >= 2 iff epsilon >= 2/3
    Also, epsilon has been defined to be strictly less than 1.
    so 2/3 = 5
    Also epsilon < 1
    iff 3epsilon < 3
    iff 3 + 3epsilon < 6
    so 5 6 in this case
    so x \in [5/3, 2) is indeed the only interval of x values that works.

  • @weo9473
    @weo9473 Před 4 měsíci +2

    1st view ❤

    • @danobro
      @danobro Před 4 měsíci +1

      2nd view :(

  • @mayankvardani9849
    @mayankvardani9849 Před 3 dny

    You should have added the initial two inequalities to make this solution shorter

  • @migssdz7287
    @migssdz7287 Před 3 měsíci

    This question becomes realy easy if you realise that floor(x) can only be 1. If it's 2 or more, floor(3x) would be at least 6 and the sum would be at least 8. If it's 0 or less, floor(3x) would be at most 3 and so would the entire sum.
    After realising that, the question is just floor(3x) = 5 -> 5 ≤ 3x < 6

  • @danielc.martin1574
    @danielc.martin1574 Před 4 měsíci +1

    Ez but cool!

  • @Jono98806
    @Jono98806 Před 4 měsíci

    Instead of solving for the k, you can just eliminate the k by adding the 2 inequalities together and then dividing throughout by 4 to solve for x.

    • @fahrenheit2101
      @fahrenheit2101 Před 3 měsíci

      No you can't. For one, division by 4 doesn't tend to leave denominators of 3...
      This is a rather key note. Just because you can simplify something into something that *looks* like an answer doesn't mean you have the answer. If you follow all the laws of logic, you just have a true statement, in this case, that x lies between 1.5 and 2. Which is true, but we can, and must, do better to get the solution set, which spans from 5/3 to 2.

    • @Jono98806
      @Jono98806 Před 3 měsíci

      @@fahrenheit2101 Yes, I realised that you have to take into account that k is an integer, especially when solving floor equations, which further restricts the solution set. So, you have to solve for k first before using the allowed integer values of k to solve for x.

  • @Pramit1156
    @Pramit1156 Před 4 měsíci +1

    This one was pretty easy

  • @marcusorban2439
    @marcusorban2439 Před 3 měsíci

    I solved it in a more intuitive way:
    1. Set x=1: the result is too small, set x=2 and the result is too big. That means the solution is something along the line of 1,...
    2. That means that floor(x) will always be 1 and floor(3x) must equal 5
    Therefore, our lower bound for the solution is 5/3, the upper bound is 6/3=2
    => x=[5/3, 2)

  • @gdmathguy
    @gdmathguy Před 4 měsíci

    There's a way easier way to solve:
    Try 1
    3+1=4
    x>1
    Try 2
    6+2=8
    x

  • @m.h.6470
    @m.h.6470 Před 4 měsíci

    Solution:
    x has to be 1 < x < 2, otherwise the left side is too small or too big.
    therefore floor(x) = 1.
    this means, that floor(3x) = 5.
    if x < 5/3, floor(3x) < 5.
    therefore 5/3 ≤ x < 2.

  • @pedroguilherme868
    @pedroguilherme868 Před 3 měsíci

    What i found before watching:
    The values need to be between one and two
    X=1 results in 3+1=4
    X=2 results in 6+2=8
    The floor of anything bwtween 1 and 2 is 1
    So the floor of x is always 1
    (Floor of 3x) +1=6
    The floor of 3x needs to be 5
    The lowest value is when 3x=5, or x=5/3, and the upper limit is when x approaches 2, since 3x would then approach but not equal 6, and thus the floor would still be 5
    Thus, x is bigger than or equal to 5/3 and smaller than 2

  • @janda1258
    @janda1258 Před 4 měsíci +3

    I’m sorry my guy, I cannot fokus on the math. That chalk is way to smooth to not admire

  • @pianoforpresident4154
    @pianoforpresident4154 Před 4 měsíci

    this whole floor thing seems like wizards of the coast invented yet another silly ability in their magic the gathering decks...

  • @oclati8313
    @oclati8313 Před 4 měsíci

    Or we can take each value of both and look when the sum equals 6 ( sry for probable english mistake french here) 👍 but ur technique seems better anyways

  • @jasonryan2545
    @jasonryan2545 Před 4 měsíci +1

    Welcome to another video ... Lets find x!